INMO 2024
Official Solutions
Problem 1. In triangle ABC with CA = CB, point E lies on the circumcircle of ABC
such that ∠ECB = 90◦ . The line through E parallel to CB intersects CA in F and AB in
G. Prove that the centre of the circumcircle of triangle EGB lies on the circumcircle of
triangle ECF .
Solution 1.
C O
C
E O
A
G B
F
O1
F O1
E
A G B
We have F G = F A since F G is parallel to BC. But also △GAE is a right angle triangle.
Thus, if F ′ is the midpoint of GE, then ∠GAF = ∠F GA = ∠F ′ GA = ∠GAF ′ which implies
F ≡ F ′ . Thus, F is the midpoint of GE.
If O is the circumcenter of △EBG, then
∠F OE = ∠GBE = ∠ABE = ∠ACE = ∠F CE.
Thus, we get ∠F OE = ∠F CE as desired.
Solution 2. (∠BCA acute case) Let O1 be the circumcenter of △ABC, O be the circum-
center of △EBG and ω be the circumcircle of △ECF .
First, show that F is the midpoint of EG as in Solution 1. Next, we show that O1 lies
on ω. This follows from
∠EO1 C = 2∠EBC = 2∠O1 BC = 2∠BCO1 = ∠BCA = ∠EF C.
Now, O1 is the midpoint of EB and F is the midpoint of EG, therefore the homothety
at E with ratio 1/2 takes △EGB to △EF O1 . Thus, it takes O, the circumcenter of △EGB,
to the circumcenter of △EF O1 , thus proving that the midpoint of EO is the center of ω.
This immediately implies that O lies on ω.
Remark.
• There are two configurations possible in the above problem, one for C acute and one
for obtuse. One may replace all the angles in the above solutions by directed angles
to obtain a solution which remains invariant in all configurations.
• There is a direct solution by first proving that O is on BC by calculating ∠EBO and
then proving that ∠OCF = ∠OEF . However, this needs more care for handling both
configurations.
1
Problem 2. All the squares of a 2024 × 2024 board are coloured white. In one move, Mohit
can select one row or column whose every square is white, choose exactly 1000 squares
in this row or column, and colour all of them red. Find the maximum number of squares
that Mohit can colour red in a finite number of moves.
Solution. Let n = 2024 and k = 1000. We claim that the maximum number of squares
that can be coloured in this way is k(2n − k), which evaluates to 3048000.
Indeed, call a row/column bad if it has at least one red square. After the first move,
there are exactly k + 1 bad rows and columns: if a row was picked, then that row and
the k columns corresponding to the chosen squares are all bad. Any subsequent move
increases the number of bad rows/columns by at least 1. Since there are only 2n rows
and columns, we can make at most 2n − (k + 1) moves after the first one, and so at most
2n − k moves can be made in total. Thus we can have at most k(2n − k) red squares.
To prove this is achievable, let’s choose each of the n columns in the first n moves, and
colour the top k cells in these columns. Then, the bottom n − k rows are still uncoloured,
so we can make n − k more moves, colouring k(n + n − k) cells in total.
Problem 3. Let p be an odd prime number and a, b, c be integers so that the integers
a2023 + b2023 , b2024 + c2024 , c2025 + a2025
are all divisible by p. Prove that p divides each of a, b, and c.
Solution 1. Set k = 2023. If one of a, b, c is divisible by p, then all of them are. Indeed,
for example, if p | a, then p | ak + bk implies p | b, and then p | bk+1 + ck+1 implies p | c. The
other cases follow similarly.
So for the sake of contradiction assume none of a, b, c is divisible by p. Then
ak(k+2) ≡ (ak )k+2 ≡ (−bk )k+2 ≡ −bk(k+2) (mod p)
and
ak(k+2) ≡ (ak+2 )k = (−ck+2 )k ≡ −ck(k+2) (mod p).
So bk(k+2) ≡ ck(k+2) (mod p). But then
2 2
ck(k+2) · c ≡ c(k+1) ≡ (−bk+1 )k+1 ≡ b(k+1) ≡ bk(k+2) · b ≡ ck(k+2) · b (mod p)
which forces b ≡ c (mod p). Thus
0 ≡ bk+1 + ck+1 = 2bk+1 (mod p)
implying p | b, a contradiction. Thus the proof is complete.
Solution 2. As before, we may assume p divides none of a, b, and c and set k = 2023.
Then
ak ≡ −bk (mod p)
k+1 k+1
b ≡ −c (mod p)
k+2 k+2
c ≡ −a (mod p)
and multiplying these three equations yields ak bk+1 ck+2 ≡ −bk ck+1 ak+2 (mod p). By can-
celling the factor ak bk ck+1 , we get a2 ≡ −bc (mod p). Now
p | ak + bk =⇒ a4k ≡ b4k (mod p) =⇒ c2k ≡ b2k (mod p)
so
p | bk+1 + ck+1 =⇒ b2(k+1) ≡ c2(k+1) (mod p) =⇒ b2 ≡ c2 (mod p)
so either b ≡ c (mod p) or b ≡ −c (mod p). In the latter case, a2 ≡ c2 (mod p) so a ≡ c
(mod p) or a ≡ −c (mod p). In any case, two out of {a, b, c} are the same mod p, so one of
the equations gives p | 2xy where x ∈ {a, b, c} and y ∈ {k, k + 1, k + 2}, hence p odd implies
p | x so p | abc, the desired contradiction.
2
Solution 3. We have
a2023 ≡ −b2023 (mod p) (1)
2024 2024
b ≡ −c (mod p) (2)
2025 2025
c ≡ −a (mod p) (3)
Thus,
a2023·2024·2025 ≡ b2023·2024·2025 (mod p) by (1)
2023·2024·2025
≡ −c (mod p) by (2)
2023·2024·2025
≡ −a (mod p) by (3)
Thus, p | 2 · a2023·2024·2025 and hence p | a since p is odd. Now, finish as before.
Remark.
• Solution 3 is the shortest, and seems to crucially relies on 2024 being even but it can
be modified to always work. In particular, instead of raising to power k(k + 1)(k + 2),
we raise it to lcm(k, k + 1, k + 2). This method even works if we have a longer chain of
equations and more variables i.e. p|ak+i i + ak+i
i+1 for all i in 0, · · · , n − 1 and an = a0 .
• There are other possible approaches: one using primitive roots and another using
orders. These proceed by considering the highest power of 2 dividing the exponents
of the primitive root or order of ab−1 mod p respectively.
Problem 4. A finite set S of positive integers is called cardinal if S contains the integer
|S|, where |S| denotes the number of distinct elements in S. Let f be a function from
the set of positive integers to itself, such that for any cardinal set S, the set f (S) is also
cardinal. Here f (S) denotes the set of all integers that can be expressed as f (a) for some
a in S. Find all possible values of f (2024).
Note: As an example, {1, 3, 5} is a cardinal set because it has exactly 3 distinct elements,
and the set contains 3.
Solution 1. The possible values are 1, 2, and 2024.
Construction. The function f (x) = 1 for all x ∈ N works. Also, f (x) = 1 for all x ̸= 2024
and f (2024) = 2, works. Finally, f (x) = x for all x ∈ N works as well.
It remains to show these are the only possible values for f (2024).
Proof. Denote Im(f ) = {f (x) | x ∈ N}. The cardinal set {1} gives f (1) = 1. Consider the
following two cases:
• Im(f ) is unbounded. Fix any n ∈ N, with n > 1. Pick n−1 distinct integers k1 , . . . , kn−1
such that f (ki ) ̸∈ {n, f (n)} and f (ki ) are all pairwise distinct, for 1 ≤ i < n. Then
{n, k1 , . . . , kn−1 } is a cardinal set. Then {f (n), f (k1 ), . . . , f (kn−1 )} is a cardinal set with
n distinct elements, so n lies in this set, hence f (n) = n. This gives the identity
function.
• Im(f ) is bounded. Suppose f (x) ≤ M for all x ∈ N and some integer M > 0.
Claim. For any integer a satisfying 1 ≤ a ≤ M , if there are infinitely many integers
n ∈ N such that f (n) = a, then a = 1.
Proof. Let b > 1 be one of the integers with f (b) = a. Consider b − 1 other integers
c1 , . . . , cb−1 , such that f (ci ) = a for 1 ≤ i < b, and ci are all pairwise distinct. Then
{b, c1 , . . . , cb−1 } is a cardinal set, so the image set, which consists of the singleton {a}
is cardinal, hence a = 1.
So for every 2 ≤ m ≤ M , there are only finitely many integers x such that f (x) = m.
Thus, there exists an integer N > 1 such that for all n ≥ N , f (n) = 1. Now for every
1 < l < N , consider the cardinal set {l, N + 1, N + 2, . . . , N + l − 1}. Then the image set
consists of {1, f (l)}, which can be cardinal only when f (l) = 1 or f (l) = 2.
By the above reasoning, f (2024) can only be 1, 2, or 2024, each of which occurs as an
example.
3
Solution 2. We present a second proof of the fact that the proposed values are the only
possibilities. Considering the singleton cardinal set {1}, we see that f (1) = 1. The cardinal
set {1, 2} gets mapped to {1, f (2)}, so f (2) must be 2 or 1.
Case 1. Suppose f (2) = 1. Now {2, 2024} is a cardinal set, and therefore so is {1, f (2024)}.
This means f (2024) is 1 or 2.
Case 2. Suppose f (2) = 2. The cardinal set f ({1, 2, 3}) = {1, 2, f (3)} shows that f (3) ∈
{1, 2, 3}, but the cardinal set f ({2, 3}) = {2, f (3)} proves f (3) cannot be 2. Thus there are
two sub-cases:
2.1. f (3) = 1. Then the set {1, 3, 2024} is cardinal, hence so is {1, f (2024)}, implying, as
before, f (2024) ∈ {1, 2}.
2.2. f (3) = 3. In this case, we show via induction that f (n) = n for all n ∈ N.
The base cases n = 1, 2, 3 are already known. Now consider n ≥ 4, and assume
f (k) = k for all k < n. Consider the cardinal f ({1, 2, . . . , n}) = {1, 2, . . . , n − 1, f (n)}
which implies f (n) ∈ {1, 2, . . . , n}.
However, consider the n − 1-element cardinal set {1, 2, . . . , n} \ {n − 2}. For its image
to be cardinal f (n) cannot equal any number in {1, 2, . . . , n − 1} \ {n − 2}; else its
cardinality would be n − 2, which isn’t in the set. So f (n) ∈ {n − 2, n}.
Finally, consider the n − 2-element set {1, 2, . . . , n} \ {n − 1, n − 3}. If f (n) = n − 2,
its image would only have n − 3 elements, and thus would not be cardinal. So we
conclude that f (n) = n and the induction is complete. In particular, f (2024) = 2024.
Thus the only possible values of f (2024) are 1, 2, and 2024.
Remark.
• There are many ways to finish Solution 2 after reaching (2.2):
1. Induct on n and get 1 ≤ f (n) ≤ n as before. Now, note that f (n) ̸= 1 for n > 3 by
considering {1, 3, n}, and then if f (n) < n we get a contradiction on considering
{1, 2, . . . , f (n) − 2, f (n), n}.
2. Show that f (n) ̸= 1, 2, 3 for n > 3 by considering {2, n} and {1, 3, n}. Then prove
that f is injective by considering {3, n, m} if n > m > 3. Now, finish by induction
and considering {1, 2, . . . , n}.
3. Suppose n0 is the smallest integer so that f (n0 ) ̸= n0 for some n0 > 3, and let t
be the smallest value achieved by f (n) for n ≥ n0 . Let f (m) = t. Then t ̸= 1, 2, 3
and t < n0 ≤ m as before. And now consider {t, m, m + 1, . . . , m + t − 2} to get that
f must take on a value smaller than t.
Problem 5. Let points A1 , A2 , and A3 lie on the circle Γ in counter-clockwise order, and
let P be a point in the same plane. For i ∈ {1, 2, 3}, let τi denote the counter-clockwise
rotation of the plane centred at Ai , where the angle of the rotation is equal to the angle
at vertex Ai in △A1 A2 A3 . Further, define Pi to be the point τi+2 (τi (τi+1 (P ))), where indices
are taken modulo 3 (i.e., τ4 = τ1 and τ5 = τ2 ).
Prove that the radius of the circumcircle of △P1 P2 P3 is at most the radius of Γ.
Solution 1. Fix an index i ∈ {1, 2, 3}. Let D1 , D2 , D3 be the points of tangency of the
incircle of triangle △A1 A2 A3 with its sides A2 A3 , A3 A1 , A1 A2 respectively.
The key observation is that given a line ℓ in the plane, the image of ℓ under the map-
ping τi+2 (τi (τi+1 (ℓ))) is a line parallel to ℓ. Indeed, ℓ is rotated thrice by angles equal to
the angles of △A1 A2 A3 , and the composition of these rotations induces a half-turn and
translation on ℓ as the angles of △A1 A2 A3 add to 180◦ . Since Di is a fixed point of this
τi+1 τ τi+2
transformation (by the chain of maps Di → Di+2 →i Di+1 → Di ), we conclude that the
line P Di maps to the line Pi Di . But the two lines are parallel and both of them pass
through Di hence they must coincide, so Di lies on P Pi . Further, each rotation preserves
distances, hence Pi is the reflection of P in Di .
In other words, the triangle P1 P2 P3 is obtained by applying a homothety with ratio 2
and center P to the triangle D1 D2 D3 . Thus, the radius of the circumcircle of △P1 P2 P3 is
twice the radius of the circumcircle of △D1 D2 D3 , i.e., twice the radius of the incircle of
△A1 A2 A3 , which is known to be at most the radius of the circumcircle Γ.
4
Solution 2. Toss the figure on the complex plane, and let A1 = a, A2 = b, A3 = c without
loss of generality. Let the angles of the triangle at A1 , A2 , A3 be denoted by A, B, C.
Now, for any complex number z, the rotation at z0 with angle θ counterclockwise sends
z to (z − z0 )eiθ + z0 .
Therefore, one computes that
τ321 (z) = τ3 (τ1 (τ2 (z))) = τ3 (τ1 ((z − b)eiB + b))
= τ3 (zei(A+B) + beiA (1 − eiB ) + a(1 − eiA ))
= −z + b + c + bei(A+C) + aeiC − aei(A+C) − ceiC
Thus, τ312 (z)+z is independent of z. Similarly τ123 (z)+z and τ231 (z)+z are also independent
of z. Note that adding z is the same as translation by z, hence we have shown that the
circumradius of △P1 P2 P3 is independent of P .
Thus, it suffices to prove the result for z = z0 = a + b + c. Let U = −τ312 (z0 ), V =
−τ123 (z0 ), W = −τ231 (z0 ). So, it is enough to prove that the circumradius of △U V W at
most the radius of Γ.
A U K
M
W
B C
V
L
Name the vertices A1 , A2 , A3 as A, B, C for convenience. Let the parallel line to BC
passing through A intersect Γ again at K. Similarly, define L as the second intersection
of the line through B parallel to CA and finally M for C parallel to AB.
We claim that U lies on the line segment AK: We have U = a − (b − a)ei(A+C) + (c − a)eiC ,
−→ −−→
hence AU is parallel to AK hence U lies on the line AK. If AB = AC then U = A,
−→
and the claim is proven. Else suppose that AB < AC. Then AU points towards K and
|AU | = AC − AB, so it suffices to show that AK > AC − AB. But this is clear because
KCBA is an isosceles trapezium, so AB = KC, and then triangle inequality on △KAC to
get KA + KC > AC.
Thus, U ∈ AK, and similarly V ∈ BL, W ∈ CM . We claim that for any U, V, W on the
segments AK, BL, CM respectively, the circumradius of △U V W is less than or equal to
the radius of Γ.
←→
Now let X, Y be two fixed points on the same side of a line ℓ. Fix a side of XY , and
←→
let Z be a variable point on ℓ which always remains on this fixed side of XY . Then the
←→
circumradius of △XY Z is minimized at the unique point Z0 (on this fixed side of XY )
for which the circumcircle of △XY Z0 is tangent to ℓ and it is a increasing function as
one goes further away from this unique point Z0 . Thus, the maximum circumradius of
△U V W is achieved only if U ∈ {A, K}, V ∈ {B, L}, W ∈ {C, M }. For each of these, the
circumradius is the radius of Γ, hence we are done.
Remark.
• The conclusion of Solution 1 used the fact that in a triangle ABC with incentre I
and inradius r, and circumcentre O and circumradius R, we have the inequality
R ≥ 2r. This is called Euler’s Inequality. The standard proof is that 0 ≤ OI 2 =
R2 − Pow(I, (O, R)) = R2 − 2Rr. The last equality holds as Pow(I, (O, R)) = IA · IM
where M is the midpoint of minor arc BC
d in the circumcircle of ABC, and because
2R sin A
IA = r A and IM = M B = a
A = A
A = 2R sin 2 by using “the trident lemma"
sin 2 2 cos 2 2 cos 2
and the double-angle sine formulas.
5
• After proving that the circumradius is independent of P , one can take P = I, for
which P1 is easily seen to be the point such that D1 is the midpoint of IP1 . Now we
again finish by Euler’s Inequality.
Problem 6. For each positive integer n ≥ 3, define An and Bn as
p p p
An = n2 + 1 + n2 + 3 + · · · + n2 + 2n − 1,
p p p
Bn = n2 + 2 + n2 + 4 + · · · + n2 + 2n.
Determine all positive integers n ≥ 3 for which ⌊An ⌋ = ⌊Bn ⌋.
Note. For any real number x, ⌊x⌋ denotes the largest integer N such that N ≤ x.
Solution. Let M = n2 + 12 n.
Lemma 1. Bn − An < 12 .
Indeed,
n p n n
X p X 1 X 1
(Bn − An ) = n2 + 2k − n2 + 2k − 1 = √ √ <
k=1 k=1
n2 + 2k + n2 + 2k − 1 2n
k=1
n 1
= =
2n 2
proving the lemma.
Lemma 2. An < M < Bn .
Proof. Observe that
n p n n
X X 2k − 1 X 2k − 1 n2
(An − n2 ) = n2 + 2k − 1 − n = √ < =
k=1 k=1
n2 + 2k − 1 + n k=1 n + n 2n
n
=
2
Pn
as k=1 (2k − 1) = n2 , proving An − n2 < n
2 or An < M . Similarly,
n p n n
X X 2k X 2k
(Bn − n2 ) = n2 + 2k − n = √ >
k=1 k=1
n + 2k + n k=1 (n + 1) + n
2
n(n + 1) n
= >
2n + 1 2
Pn
as k=1 (2k) = n(n + 1), so Bn − n2 > n
2 hence Bn > M , as desired.
By Lemma 2, we see that An and Bn are positive real numbers containing M between
them. When n is even, M is an integer. This implies ⌊An ⌋ < M , but ⌊Bn ⌋ ≥ M , which
means we cannot have ⌊An ⌋ = ⌊Bn ⌋.
When n is odd, M is a half-integer, and thus M − 12 and M + 12 are consecutive integers.
So the above two lemmas imply
1 1
M− < Bn − (Bn − An ) = An < Bn = An + (Bn − An ) < M + .
2 2
This shows ⌊An ⌋ = ⌊Bn ⌋ = M − 12 .
Thus, the only integers n ≥ 3 that satisfy the conditions are the odd numbers and all
of them work.